Solve the following:


2x-7\4 = x+3/3


Hope u get this!

Answers

Answer 1

Answer:

hello it is so

Step-by-step explanation:

simple simply do the division if don't I can

Answer 2
The correct answer is X=33/2

Related Questions

Khairul is x years old .Khairul`s uncle is 4 times as old as khairul will be in 5 years .Find the present age of Kharuls`s uncle. Plz some one answer this question will be very thankfull

Answers

Answer:

4x + 15

Step-by-step explanation:

Let the age of Khairul's uncle y.

From the question given above, the following were obtained:

Khairul's age = x years

In 5 years time:

Khairul's age = x + 5

Khairul's uncle age = y + 5

But from the question given above,

Khairul`s uncle will be 4 times as old as what khairul will be in 5 years. This can be written as follow:

y + 5 = 4(x + 5)

Clear bracket

y + 5 = 4x + 20

Subtract 5 from both side

y + 5 – 5 = 4x + 20 – 5

y = 4x + 15

Therefore, Khairul's uncle present age is 4x + 15

Answer:

y= 4x+15

Step-by-step explanation:

This problem tests our ability to express word problem using equation

Given data

Khairul age = x years

let the uncle's age be y

in five years time, Khairul will be = x+5

in five years time, the uncle will be = y+5

 

hence Khairul uncle's age  is  

y= 4(x+5)

y=4(x+5)

y=4x+20

hence the uncles current age is

y+5=4x+20

y= 4x+20-5

y= 4x+15

   

Help ASAP !!!!!!!!!!!!!!!!!!!

Answers

Answer:

im lik 999999999

percent sure its 2

Step-by-step explanation:

Answer:

divide r by s and multiply by 3   :3   :8    xD

Step-by-step explanation:

20. Rolando drives at least 40 miles but less than 60 miles each week.
Part A. Write and then graph the compound inequality representing all of the possible distances Rolando could
drive for 8 weeks. (7 points)

Answers

Answer:

Step-by-step explanation:

60 miles

How could you prove that a number is divisible by 16? Use the space below to test your hypothesis using the numbers 144 & 256. Explain your new rule.

Answers

Answer:

We factor the number 16 into prime numbers to get their factors.

We do the same with any given number. Only if it has the same factor as 16 that numberwill be divisible by 16

Both numbers proposed (144&256) are divisible by 16

Step-by-step explanation:

To prove it we need to factor the number 16 to get their expression in prime numbers:

16     2

 8    2

 4    2

 2    2

 1

16 is equal to [tex]2^{4}[/tex]

We have to factor a number and if their factor include [tex]2^{4}[/tex] or a higher power

then, they are divisible by 16

144        2

 72       2

 36       2

 18        2

   9       3

   3       3

   1

144 is queal to:  [tex]2^4 . 3^2[/tex] as it does have [tex]2^4[/tex] It is divisible by 16

256         2

128          2

  64        2

  32        2

   16        2

    8        2

    4        2

    2        2

    1

256 is equal to [tex]2^8[/tex] This contains [tex]2^4[/tex] as [tex]2^8 = 2^4 . 2^4[/tex]

therefore it is divisible by 16

The figure shows two triangles on a coordinate grid: A coordinate grid is shown from positive 6 to negative 6 on the x-axis and from positive 6 to negative 6 on the y-axis. A triangle ABC is shown with vertex A on ordered pair negative 4, 1 vertex B on ordered pair negative 3,1, and vertex C on ordered pair negative 4, 4. Another triangle A prime B prime C prime is shown with vertex A prime on ordered pair 4, 4, vertex B prime on ordered pair 3, 4, and vertex C prime on ordered pair 4,1. What set of transformations is performed on triangle ABC to form triangle A′B′C′? (5 points) A 180-degree counterclockwise rotation about the origin, followed by a translation 5 units down A translation 5 units down followed by a 180-degree counterclockwise rotation about the origin A 270-degree counterclockwise rotation about the origin, followed by a translation 5 units to the right A translation 5 units to the right, followed by a 270-degree counterclockwise rotation about the origin

Answers

Answer:

  A translation 5 units down followed by a 180-degree counterclockwise rotation about the origin

Step-by-step explanation:

The figure is obviously rotated 180°. If the rotation is first, the image would be in the 4th quadrant, where it would stay after translation downward.

If the translation is downward first, it would be in the 3rd quadrant, moving to the first quadrant after rotation by 180°. This is the apparent sequence of transformations:

  translation down 5, rotation 180°

Answer:

translation down 5, rotation 180°

Step-by-step explanation:

Which of the folloving statements is the most precise and corred to define perpendicular lines?

Answers

Answer:C

Step-by-step explanation:

Which answer describes the type of numbers that are dense? whole numbers and integers whole numbers but not integers rational numbers and irrational numbers rational numbers but not irrational numbers

Answers

Answer -rational numbers and irrational numbers

Using number sets, it is found that the set of dense numbers is composed of rational and irrational numbers. Therefore, option C is the correct answer.

From options, we need to check which answer describes the type of numbers that are dense.

What are numbers?

Numbers are an integral part of our everyday lives, right from the number of hours we sleep at night to the number of rounds we run around the racing track and much more. In math, numbers can be even and odd numbers, prime and composite numbers, decimals, fractions, rational and irrational numbers, natural numbers, integers, real numbers, rational numbers, irrational numbers, and whole numbers.

Numbers can be classified as:

Whole numbers: All positive numbers and 0, so: {0,1,2,...}

Integer numbers: Positive or negative, not decimal so: {...,-2,-1,0,1,2,....}

Rational numbers: Integer plus decimals that can be represented by fractions, that is, they either have a pattern or have a finite number of decimal digits, for example, 0, 2, 0,45(finite number of decimal digits), 0.3333(3 repeating is the pattern), 0.32344594459 (4459 repeating is the pattern).

Irrational numbers: Decimal numbers that are not represented by patterns, that is, for example, 0.1033430290339, which can be approximated to the rational 0.1.

Real numbers: Rational plus irrational.

A subset is dense of the rational numbers if all numbers can be approximated to rational numbers, and thus, rational and irrational numbers are dense.

Therefore, option C is the correct answer.

To learn more about the rational and irrational visit:

https://brainly.com/question/15267867.

#SPJ6

5x
2x
7x
4x
Equation
X=

Answers

Answer:

look this for the solution

Is due today Help!!! Algebra ll

Answers

[tex]-7x-9y=-8[/tex]

[tex]5x+6y=4[/tex]

[tex]-35x-45y=-40[/tex]

[tex]35x+42y=28[/tex]

[tex]-3y=-12[/tex]

[tex]y=4[/tex]

[tex]5x+6(4)=4[/tex]

[tex]5x+24=4[/tex]

[tex]5x=-20[/tex]

[tex]x=-4[/tex]

[tex](-4,4)[/tex]

Hope this helps.

頑張って!

2 ABC Company has a large piece of equipment
that cost $85,600 when it was first purchased 6
years ago. The current value of the equipment is
$30,400. What is the average depreciation of the
equipment per year?
F. $ 5,800
G. $ 9,200
H. $15,200
J. $27,600
K. $42,800

Answers

Answer:5,800

Step-by-step explanation:

The average depreciation of the equipment per year is $9200.

What is depreciation?

The monetary value of an asset decreases over time due to use, wear and tear or obsolescence, this is called as depreciation.

Now it is given that,

Cost of the equipment = $85,600

current value of the equipment is given as $30,400

Duration for which the equipment was used = 6 years

Now the depreciation of the equipment is given as,

Total depreciation in the price of the equipment

                       = Cost of the equipment - current price of the equipment

So, total depreciation = $85,600 - $30,400 = $55,200

Now,

Average yearly depreciation = Total depreciation ÷ Years used

Average depreciation = 55,200/6

⇒Average depreciation = $9200

Therefore, average depreciation of the equipment per year is $9200.

More about depreciation :

https://brainly.com/question/9434632

#SPJ5

Evaluate this question (2/5^3)

Answers

Answer:

The answer will be 18/125 and the decimal form is 0.064

Step-by-step explanation:

Write a compound inequality that represents the sentence below.
A quantity x is either less than 2 or greater than 16.

Answers

Answer:

2<x>16

Step-by-step explanation:

What is the simplified form of 3a^2+6a+2b^2.

It would be greatly appreciated if anyone could solve this

Answers

Answer:

3 a (a + 2) + 2 b^2 or a (3 a + 6) + 2 b^2 or 3 (a + 1)^2 + 2 b^2 - 3

Step-by-step explanation:

GHKL is a rectangle. which of the following is a false statement?

Answers

Answer:

The false statement is ∠1 ≅ ∠2.

Nela is filling her circular backyard swimming pool. The radius of the pool is 5 feet and the height is 6 feet. Which shows the correct substitution of these values into the formula for the volume of a cylinder?

Answers

Answer:

V=[tex]\pi[/tex]5^2*6

Step-by-step explanation:

If this makes it a little easier to read:

V = [tex]\pi[/tex] * 5^2 * 6

This is the answer when you solve it btw:

471.24

Also, who buys a swimming pool in the shape of a cylinder?

The volume of the cylinder is 471 square feet.

Given,

Nela is filling her circular backyard swimming pool.

The radius of the pool is 5 feet and the height is 6 feet.

We need to find the volume of the cylinder.

What is the volume of a cylinder?

It is given by:

V =  [tex]\pi[/tex] r² h.

Find the volume of the cylinder

V =  [tex]\pi[/tex] r² h

We have,

h = 6 feet and r = 5 feet

V = 3.14 x 5² x 6

V = 3.14 x 25 x 6

V = 471 square feet.

Thus the volume of the cylinder is 471 square feet.

Learn more about finding the volume of a cylinder here:

https://brainly.com/question/15002580

#SPJ2

this is due tm please help​

Answers

Answer:

25. No slope

Step-by-step explanation:

25. Y2 - Y1 / X2 - X1

= -8 - (-3) / 9 - 9

= -5 / 0

= no slope; straight line

Use this formula (Y2 - Y1 / X2 - X1) for each ques.

quadrilateral A can be rotated into position of quadrilateral B. Estimate the angle of rotation​

Answers

Answer:

The angle of rotation is 120°

Step-by-step explanation:

The given quadrilateral A can be taken as being upright form, however, by measurement, using a protractor application, the base of the quadrilateral is inclined at an angle of 13° to the horizontal, the positive direction of the x-axis, before rotation while the base of the B which is the image of A after rotation, is found to be at angle of 133° to the positive direction of the horizontal x-axis

Therefore, the angle of rotation = 133° - 13° = 120°

list the numbers in order from greatest to least -1. 7, -15, -7, 2, 1

Answers

7,2,1,-1,-7,-15 that’s greatest to least

I have a math question g=mB-2qB and I have to solve for B​

Answers

Answer:

[tex]B = \frac{g}{m-2g}[/tex]

Step-by-step explanation:

Given

[tex]g = mB - 2qB[/tex]

Required

Solve for B

[tex]g = mB - 2qB[/tex]

Factorize the expression

[tex]g = B(m - 2q)[/tex]

Divide both sides by m - 2g

[tex]\frac{g}{m-2g} = \frac{B(m - 2q)}{m-2g}[/tex]

[tex]\frac{g}{m-2g} = B[/tex]

Reorder

[tex]B = \frac{g}{m-2g}[/tex]

HELP ME PLS LOVE U GUYS ,<322

Answers

Answer:

384

Step-by-step explanation:

Triangle sides: 12x10/2. Then multiply by four to get all four sides.

Bottom: 12x12.

Add together, get 384.

there were 8 lunchboxes on the first table.there were 6 lunchboxes on the second table. how many more lunchboxes were on the first table then on the second table?​

Answers

Answer:

2 lunchboxes more.

Step-by-step explanation:

Its 2 lunchboxes more as the question asked how many MORE lunchboxes the first table has then the second table so its bassically 8 - 6 = 2

Help me do the ones marked in red.

Answers

Answer:

  2. "two thirds of the fourth power of r"

  4.  n + 14

  6.  7 +11n

  8.  (2/5)n^2

__

  7.  28

  8.  1/5

Step-by-step explanation:

2. "two thirds of the fourth power of r"

__

4. "sum" means the listed items are added. You can use any convenient variable to represent "a number." I often use n for number.

  n + 14

__

6. "7 more than" means 7 is added to whatever follows. "11 times a number" means the number is multiplied by 11.

  7 + 11n

__

8. "of" means "times". The square of a number is that number to the power of 2.

  (2/5)n^2

__

7 & 8 (evaluation). Your calculator can do this. (If not, get one that can.) Note that parentheses are needed around numerators and denominators and anything else that must be treated as a single value.

Seismosaurus is the longest known dinosaur. It
measured 1800 inches. How far would 3000
Seismosaurus dinosaurs span if they were
placed head to tail? Write your answer in
scientific notation.

Answers

Answer:

5.4 * 10^6 in.

Step-by-step explanation:

3000 * 1800 in. = 5,400,000 in. = 5.4 * 10^6 in.

5. Which of the following describe the transformation rule (x, y) – (x + 1, - 2)
Move left two, down one
Move right one, down two
Move right two, down one
Move left two, up one

Answers

Answer:

shift to the right 1 unit, shift down 2 units

Step-by-step explanation:

because x+1 which mean shift to the right 1 unit as +1 is positive

down 2 because y is -2 as well as shift down 2 units

Solve the following equation for X... -3x + 12 = 24​

Answers

Answer:

the answer is -4

Step-by-step explanation:

-3x+12=24

     -12 -12

-----------------

     -3x=12

     12 divided by-3= -4

x= -4

Answer:

x=-4

Step-by-step explanation:

move the constant

-3x=24-12

-3x=12

divide both side by -3

ANSWER=-4

Alan's go kart travels 1750 feet per minute and Barry's go kart travels 21 miles per hour whose go kart travels faster round your answer to the nearest tenth

Answers

Answer:

Barry's kart is faster as he goes 21 Mph while Alan only goes 19.89 Mph.

Step-by-step explanation:

First to get this answer we need to convert feet per minute to miles per hour. So we multiple 1750 feet per minute by 60 to get a hour. This gets up 105000 total feet in a hour. Now we need to divide this by 5280, which is how long a mile is, to get 19.89 miles which is how many miles he travels in a hour making him travel 19.89 mph which is less than 21 making Barry faster.

oooooooooooooooooooooooooooooooooooooooooooooooooooo

Answers

Oooooooooooooooooooooooooooooooooooooooooooooooooooooooooooooooo
Ahhhhhhhhhhhhhhhhhhhhhhhh

Find the area and perimeter of the figure below.

Answers

Answer:

Area: 6x+6

Perimiter: 4x+10

Step-by-step explanation: to find the area of something multiply the width times the height. to find the perimeter of something add each side together. use the order of operations (PEMDAS).

Area=3(2x+2)

Area=6x+6

Perimeter=3+3+(2x+2)+(2x+2)

Perimeter=6+4x+4

Perimeter=4x+10

Answer:

Area=length*width

A=3(2x+2)=6x+6 unit squar

Perimeter = 2l+2w

P=2(3)+2(2x+2)

P=6+4x+4

P=10+4x

Please help and show work! Simplify each expression!




-7y - (-12y) =



15xy- (-6xy) =



-53va - 32va



Answer the expressions and show work please!



64 / 0.8 -5.6 / 7 =



Use the distributive property to simplify the expression. Show work!



-(x -3) + 6 =



Use order of operations to evaluate the expression.



Show work please!



6 + 3 (9) =



Evaluate the expressions if x = 10, y = 5, and z = 1.


x/y =



xy + z



5 (z - x)



Please answer all of these questions correctly and show work! I really need this! I'm putting 50 points and I'll choose brainliest!

Answers

Answer:

See below

Step-by-step explanation:

-7y - (-12y) =  -7y + 12y = (-7+12)y = 5y  15xy- (-6xy) =  15xy + 6xy = (15+6)xy = 21xy-53va - 32va  = (-53 -32)va = - 85va64 / 0.8 - 5.6 / 7 = 640/8 - 56/7×1/10 = 80 - 8/10 = 80 - 0.8 = 79.2-(x -3) + 6 = -x + 3 + 6 = -x + 96 + 3 (9) = 6 + 27 = 33

Evaluate the expressions if x = 10, y = 5, and z = 1.

x/y =  10/5 = 2xy + z  = 10×5 + 1 = 50 + 1 = 515 (z - x) = 5×(1 - 10) = 5×(-9) = -45

Answer:

see below

Step-by-step explanation:

⇒  -7y - (-12y) = -7y + 12y =  5y

⇒  15xy - (-6xy) = 15xy + 6xy = 21xy

⇒  -53va - 32va =    - 85av

⇒  64 / 0.8 -5.6 / 7

   = 448 - 7.48

          5*6

    = 79.2

⇒    -(x -3) + 6

   = -x + 3 + 6

   = -x + 9

⇒  6 + 3 (9)

   = 6 + 27

   = 33

Evaluate the expressions if x = 10, y = 5, and z = 1.

⇒  x/y = 10 / 5 = 2

⇒  xy + z = 10*5 + 1 = 51

⇒  5 (z - x) = 5(1 - 10) = 5 - 50 = 45

will someone help with homework

Answers

i believe the answer is -79.31!
i got $-61.58 for her bank total
Other Questions
For your goal what is 1.834597/26881 Which tribes were Plains Indians? Select all that apply. A. Hopewell B. Nez Perce C. Sioux D. Comanche E. Blackfoot HELP ME PLZ ! Solve for x the slope-intercept form of an equation is y = mx + b. rewrite the equation in terms of the slope m What must you do in Disk Management so that File Explorer can recognize and use a new hard disk? Consider the function. f(x) = -2/3x - 24 Which conclusions can be drawn about f^-1(x)? Select two options. f^-1(x) has a slope of -2/3f^-1(x) has a slope of . f^1(x) has a restricted domain.f^-1(x) has a y-intercept of (0, 36).f^-1(x) has an x-intercept of (36, 0).f^-1(x) has a range of all real numbers. Measurements made while observing a plant grow 3 cm over a two week period are called: If the purpose of your message is to ask local businesses to sponsor your event, your desired outcome would most likely be The angle of elevation of a flagpole from the foot of a building is 65, what is the angle of depression of the building from the Flagpole? A certain ore is 20.7% nickel by mass. How many kilograms of this ore would you need to dig up to have 70.0 g of nickel? In an electrically neutral atom,o the number of protons equals the number ofneutrons.the number of protons equals the number ofelectrons.the number of neutrons equals the number ofelectrons. in 1703 pope gregory vii issued a decree forbidding kings from appointing high ranking? For the points (5,4)and (7,8), find the distance between the points and find themidpoint of the segment. Questions:1. How is a cell membrane like the plastic bag in the experiment?2. Define diffusion.3. List two reasons why molecules will not diffuse into cells. I NED HELP FAST! Due tomorrow!! Ways to represent relations! one of the five large bodiesof salt water that covermuch of the earth'ssurface what is 3.825 as a fraction with a guide? What is the value of 3x2+y3 when x=2 and y=2? Do the points shown represent additive inverses? Explain why or why not. What mixed number is equivalent to 12/10